Difference between revisions of "2006 AIME A Problems/Problem 15"

(Solution)
(See also)
Line 6: Line 6:
  
 
== See also ==
 
== See also ==
*[[2006 AIME II Problems]]
+
{{AIME box|year=2006|n=II|num-b=14|after=Final Problem}}
  
 
[[Category:Intermediate Algebra Problems]]
 
[[Category:Intermediate Algebra Problems]]

Revision as of 15:58, 25 September 2007

Problem

Given that a sequence satisfies $x_0=0$ and $|x_k|=|x_{k-1}+3|$ for all integers $k\ge 1,$ find the minimum possible value of $|x_1+x_2+\cdots+x_{2006}|.$

Solution

This problem needs a solution. If you have a solution for it, please help us out by adding it.

See also

2006 AIME II (ProblemsAnswer KeyResources)
Preceded by
Problem 14
Followed by
Final Problem
1 2 3 4 5 6 7 8 9 10 11 12 13 14 15
All AIME Problems and Solutions